Identificarse Registrarse

Psu
Enseñanza Básica
Enseñanza Media
Universidad
Olimpiadas
Comunidad



 
Reply to this topicStart new topic
> APMO 2000, Ssp: 3,4,5
Luffy
mensaje May 22 2007, 11:42 PM
Publicado: #1


Dios Matemático Supremo
Ícono de Grupo

Grupo: Usuario FMAT
Mensajes: 556
Registrado: 16-August 06
Desde: Rio de Janeiro
Miembro Nº: 1.950
Nacionalidad:
Colegio/Liceo: Instituto Nacional
Universidad: Instituto Nacional de Matematica Pura e Aplicada (IMPA)
Sexo:



TEX: \noindent\underline{$Problema\ 1$} Calcule la suma:<br />$\displaystyle\sum_{i=0}^{101} \dfrac{x_i^3}{1-3x_i+3x_i^2}$ para $x_i=\dfrac{i}{101}$ .

Solucion:


TEX: \noindent\underline{$Problema\ 2$} Encuentre todas las permutaciones $a_1,...,a_9$ de $1,...,9$ tales que <br />\\ $a_1+a_2+a_3+a_4=a_4+a_5+a_6+a_7= a_7+a_8+a_9+a_1$ <br />\\y<br />\\$a_1^2+a_2^2+a_3^2+a_4^2=a_4^2+a_5^2+a_6^2+a_7^2= a_7^2+a_8^2+a_9^2+a_1^2$

Solucion:


TEX: \noindent\underline{$Problema\ 3$} Sea $ABC$ un triangulo. Sean $M$ y $N$ los puntos en los cuales la transversal de gravedad y la bisectriz desde $A$, respectivamente, cortan a $BC$. Sean $Q$ y $P$ los puntos en los que la perpendicual por $N$ a $NA$ corta a $MA$ y $BA$ respectivamente. Y $O$ el punto en el cual la perpendicular a $BA$ por $P$ encuentra a $AN$.<br /><br />Pruebe que $QO$ es perpendicular a $BC$.

Solucion: (Pendiente)

TEX: \noindent\underline{$Problema\ 4$} Sean $n,k$ enteros positivos con $n>k$ . Pruebe que: <br />$\dfrac{1}{n+1} \cdot \dfrac{n^n}{k^k (n-k)^{n-k}} < \dfrac{n!}{k! (n-k)!} < \dfrac{n^n}{k^k(n-k)^{n-k}}$

Solucion: (Pendiente)

TEX: \noindent\underline{$Problema\ 5$} Dada una permutacion ($a_0,a_1,...,a_n$) de la secuencia $0,1,...,n$.  Una transportacion de $a_i$ con $a_j$ el llamada legal si $a_i=0$ para $i>0$ , y $a_{i-1}+1=a_j$. La permutacion ($a_0,a_1,...,a_n$) es llamada regular si despues de un numero legal de transportaciones se vuelve ($0,1,...,n$). Diga para que numeros $n$ la permutacion ($1,n,n-1,...,3,2,0$) es regular.

Solucion: (Pendiente)

Mensaje modificado por Luffy el Sep 26 2011, 11:05 PM
Go to the top of the page
 
+Quote Post
Luffy
mensaje Mar 7 2008, 09:20 PM
Publicado: #2


Dios Matemático Supremo
Ícono de Grupo

Grupo: Usuario FMAT
Mensajes: 556
Registrado: 16-August 06
Desde: Rio de Janeiro
Miembro Nº: 1.950
Nacionalidad:
Colegio/Liceo: Instituto Nacional
Universidad: Instituto Nacional de Matematica Pura e Aplicada (IMPA)
Sexo:



TEX: $\boxed{Sp_1}$

TEX: \noindent $\mathcal{S}=\displaystyle\sum_{i=0}^{101}\dfrac{x_i^3}{1-3x_i+3x_i^2}$\\<br />\\<br />$\mathcal{S}=\displaystyle\sum_{i=0}^{101}\dfrac{x_i^3}{(1-x_i)^3+x_i^3}$\\<br />\\<br />$\mathcal{S}=\displaystyle\sum_{i=0}^{101}\dfrac{i^3}{(101-i)^3+i^3}$\\<br />\\<br />$\mathcal{S}=\left( \dfrac{0^3}{101^3+0^3}+\dfrac{101^3}{0^3+101^3}\right)+\left( \dfrac{1^3}{100^3+1^3}+\dfrac{100^3}{1^3+100^3}\right)+...+\left(\dfrac{50^3}{51^3+50^3}+\dfrac{51^3}{50^3+51^3}\right)$\\<br />\\<br />$\mathcal{S}=\displaystyle\sum_{i=0}^{50}1$\\<br />\\<br />$\boxed{\mathcal{S}=51}$

aportacion.gif
Go to the top of the page
 
+Quote Post
~Fatal_Collapse~
mensaje Jan 26 2009, 02:27 PM
Publicado: #3


Dios Matemático Supremo
Ícono de Grupo

Grupo: Usuario FMAT
Mensajes: 1.564
Registrado: 12-November 07
Desde: La Union, XIV Region de los Rios
Miembro Nº: 12.607
Nacionalidad:
Colegio/Liceo: Deutsche Schule
Universidad: Universidad Catolica de Chile
Sexo:



Problema 2:

TEX: Llamemos $A, B, C$ a los numeros ubicados en los vertices del triangulo formado. Sean $a_{1}, a_{2}$ los numeros que pertenecen a la recta de vertices $B,C$. Analogamente definimos $b_{1}, b_{2}, c_{1}, c_{2}$.<br /><br />Tenemos que $A+c_{1}+c_{2}+B=B+a_{1}+a_{2}+C=C+b_{1}+b_{2}+A=m$. Sumando: $A+B+C+45=3m\Rightarrow A+B+C\equiv 0\pmod3$ (*)<br /><br />Por otra parte $A^2+c_{1}^2+c_{2}^2+B^2=B^2+a_{1}^2+a_{2}^2+C^2=C^2+b_{1}^2+b_{2}^2+A^2=n$. Sumando: $A^2+B^2+C^2+285=3n\Rightarrow A^2+B^2+C^2\equiv 0\pmod 3$ (**)<br /> <br />Ocupando (*) y (**) podemos verificar que $A\equiv B\equiv C\equiv r\pmod 3$. De esto podemos deducir que $\boxed {(A,B,C)=(1,4,7)}$, $\boxed {(A,B,C)=(2,5,8)}$ o $\boxed {(A,B,C)=(3,6,9)}$. Analizaremos cada caso: <br /><br />Caso 1: $(A,B,C)=(1,4,7)$. Tenemos que $m=(A+B+C+45)/3=19$, donde $m$ es la suma de los numeros en cada lado del triangulo. Luego $a_{1}+a_{2}=8, b_{1}+b_{2}=11, c_{1}+c_{2}=14$. Por otra parte $n=(A^2+B^2+C^2+285)/3=117$ donde $n$ es la suma de los cuadrados de los numeros en cada lado del triangulo. Luego $a_{1}^2+a_{2}^2=52, b_{1}^2+b_{2}^2=67, c_{1}^2+c_{2}^2=100$ Pero $a_{1}, a_{2}, b_{1}, b_{2}$ son irracionales. Por lo tanto descartamos este caso.<br /><br />Caso 2: $(A,B,C)=(2,5,8)$. Tenemos que $m=(A+B+C+45)/3=20$ y $n=(A^2+B^2+C^2+285)/3=126$. Luego $a_{1}+a_{2}=7, b_{1}+b_{2}=10, c_{1}+c_{2}=13$; $a_{1}^2+a_{2}^2=37, b_{1}^2+b_{2}^2=58, c_{1}^2+c_{2}^2=97$. Obtenemos que $a_{1}=1, a_{2}=6, b_{1}=3, b_{2}=7, c_{1}=4, c_{2}=9$<br /><br />Caso 3: $(A,B,C)=(3,6,9)$. Tenemos que $m=(A+B+C+45)/3=21$ y $n=(A^2+B^2+C^2+285)/3=137$. Luego $a_{1}+a_{2}=6, b_{1}+b_{2}=9, c_{1}+c_{2}=12$; $a_{1}^2+a_{2}^2=20, b_{1}^2+b_{2}^2=47, c_{1}^2+c_{2}^2=92$, el cual no nos sirve porque $b_{1}, b_{2}, c_{1}, c_{2}$ son irracionales <br /><br />Por lo tanto podemos distribuir los numeros con el siguiente algoritmo:<br /><br />i) Los numeros en los vertices deben ser $2,5,8$.<br /><br />ii) Entre el $2$ y el $5$ deben estar el $4$ y el $9$.<br /><br />iii) Entre el $2$ y el $8$ deben estar el $3$ y el $7$<br /><br />iv) Entre el $5$ y el $8$ deben estar el $1$ y el $6$

avioncito.gif


--------------------
Ricardo Vargas Obando
Ex-alumno Deutsche Schule La Unión (Generación 2010, de los 150 años).
Novato de Licenciatura en Matemática/Estadística, en la Pontificia Universidad Católica de Chile.




Grupo de facebook de Novatos Matemática y Estadística PUC 2011

Currículum Olímpico:
  • "What we learned as children, that one plus one equals two, we know to be false. One plus one
    equals one. We even have a word when you plus another, equals one. That word is love."

  • "Todos piensan en cambiar el mundo, pero nadie piensa en cambiarse a sí mismo."
Go to the top of the page
 
+Quote Post
Luffy
mensaje Sep 26 2011, 11:03 PM
Publicado: #4


Dios Matemático Supremo
Ícono de Grupo

Grupo: Usuario FMAT
Mensajes: 556
Registrado: 16-August 06
Desde: Rio de Janeiro
Miembro Nº: 1.950
Nacionalidad:
Colegio/Liceo: Instituto Nacional
Universidad: Instituto Nacional de Matematica Pura e Aplicada (IMPA)
Sexo:



CITA(Vargüitas DSLU @ Jan 26 2009, 03:27 PM) *
Problema 2:

TEX: Llamemos $A, B, C$ a los numeros ubicados en los vertices del triangulo formado. Sean $a_{1}, a_{2}$ los numeros que pertenecen a la recta de vertices $B,C$. Analogamente definimos $b_{1}, b_{2}, c_{1}, c_{2}$.<br /><br />Tenemos que $A+c_{1}+c_{2}+B=B+a_{1}+a_{2}+C=C+b_{1}+b_{2}+A=m$. Sumando: $A+B+C+45=3m\Rightarrow A+B+C\equiv 0\pmod3$ (*)<br /><br />Por otra parte $A^2+c_{1}^2+c_{2}^2+B^2=B^2+a_{1}^2+a_{2}^2+C^2=C^2+b_{1}^2+b_{2}^2+A^2=n$. Sumando: $A^2+B^2+C^2+285=3n\Rightarrow A^2+B^2+C^2\equiv 0\pmod 3$ (**)<br /> <br />Ocupando (*) y (**) podemos verificar que $A\equiv B\equiv C\equiv r\pmod 3$. De esto podemos deducir que $\boxed {(A,B,C)=(1,4,7)}$, $\boxed {(A,B,C)=(2,5,8)}$ o $\boxed {(A,B,C)=(3,6,9)}$. Analizaremos cada caso: <br /><br />Caso 1: $(A,B,C)=(1,4,7)$. Tenemos que $m=(A+B+C+45)/3=19$, donde $m$ es la suma de los numeros en cada lado del triangulo. Luego $a_{1}+a_{2}=8, b_{1}+b_{2}=11, c_{1}+c_{2}=14$. Por otra parte $n=(A^2+B^2+C^2+285)/3=117$ donde $n$ es la suma de los cuadrados de los numeros en cada lado del triangulo. Luego $a_{1}^2+a_{2}^2=52, b_{1}^2+b_{2}^2=67, c_{1}^2+c_{2}^2=100$ Pero $a_{1}, a_{2}, b_{1}, b_{2}$ son irracionales. Por lo tanto descartamos este caso.<br /><br />Caso 2: $(A,B,C)=(2,5,8)$. Tenemos que $m=(A+B+C+45)/3=20$ y $n=(A^2+B^2+C^2+285)/3=126$. Luego $a_{1}+a_{2}=7, b_{1}+b_{2}=10, c_{1}+c_{2}=13$; $a_{1}^2+a_{2}^2=37, b_{1}^2+b_{2}^2=58, c_{1}^2+c_{2}^2=97$. Obtenemos que $a_{1}=1, a_{2}=6, b_{1}=3, b_{2}=7, c_{1}=4, c_{2}=9$<br /><br />Caso 3: $(A,B,C)=(3,6,9)$. Tenemos que $m=(A+B+C+45)/3=21$ y $n=(A^2+B^2+C^2+285)/3=137$. Luego $a_{1}+a_{2}=6, b_{1}+b_{2}=9, c_{1}+c_{2}=12$; $a_{1}^2+a_{2}^2=20, b_{1}^2+b_{2}^2=47, c_{1}^2+c_{2}^2=92$, el cual no nos sirve porque $b_{1}, b_{2}, c_{1}, c_{2}$ son irracionales <br /><br />Por lo tanto podemos distribuir los numeros con el siguiente algoritmo:<br /><br />i) Los numeros en los vertices deben ser $2,5,8$.<br /><br />ii) Entre el $2$ y el $5$ deben estar el $4$ y el $9$.<br /><br />iii) Entre el $2$ y el $8$ deben estar el $3$ y el $7$<br /><br />iv) Entre el $5$ y el $8$ deben estar el $1$ y el $6$

avioncito.gif


Solucion correcta winner_1st.gif felicitaciones victory.gif
Go to the top of the page
 
+Quote Post
asdayuyi
mensaje Feb 20 2015, 11:03 AM
Publicado: #5


Maestro Matemático
Ícono de Grupo

Grupo: Usuario FMAT
Mensajes: 81
Registrado: 10-November 12
Miembro Nº: 112.735
Nacionalidad:
Colegio/Liceo: Liceo Carmela Carvajal de Prat
Universidad: Instituto Nacional de Matematica Pura e Aplicada (IMPA)
Sexo:



P3

Sea K la interseccion de PQ con AC, es facil ver que OK es perpendicular a AC. Sea X la interseccion de la bisectriz desde A con el circuncirculo del triangulo ABC. Notemos que XM es perpendicular a BC y si trazamos las paralelas a PO y OK por X con D,E los puntos de interseccion con las rectas AB y AC respectivamente tenemos que los puntos D, M, E por Simson son colineales. Luego PK y DE son paralelas de donde por thales AP/AD=AQ/AM=AO/AX, luego QO es paralela a MX y tenemos lo pedido huglove.gif
Go to the top of the page
 
+Quote Post

Reply to this topicStart new topic
1 usuario(s) está(n) leyendo esta discusión (1 invitado(s) y 0 usuario(s) anónimo(s))
0 miembro(s):

 

Versión Lo-Fi Fecha y Hora actual: 23rd November 2024 - 06:31 PM